중앙 한계 정리와 다수의 법칙에 동의하지 않는 경우


19

이것은 본질적으로 math.se에서 찾은 질문 의 복제이며 , 원하는 답변 을 얻지 못했습니다.

보자 {Xi}iN 함께 독립적으로 동일하게 분산 된 랜덤 변수들의 시퀀스 일 E[Xi]=1V[Xi]=1 .

의 평가를 고려

limnP(1ni=1nXin)

불평등 사건의 양쪽이 무한대 인 경향이 있기 때문에이 표현은 조작되어야한다.

A) SUBTRACTION 시도

제한 문구를 고려하기 전에 양쪽에서 을 빼십시오 .n

limnP(1ni=1nXinnn)=limnP(1ni=1n(Xi1)0)=Φ(0)=12

CLT에 의한 마지막 동등성. 여기서 는 표준 정규 분포 함수입니다.Φ()

B) 멀티 플레이 시도

양변에 \ lim_ {n \ to \ infty} \ mathbb {P} \ left (\ frac {1} {\ sqrt {n}} \ cdot \ frac {1} {\ sqrt를 곱하십시오. {n}} \ sum_ {i = 1} ^ n X_i \ leq \ frac {1} {\ sqrt {n}} \ cdot \ sqrt {n} \ right) = \ lim_ {n \ to \ infty} \ mathbb {P} \ left (\ frac {1} {n} \ sum_ {i = 1} ^ n X_i \ leq 1 \ 오른쪽)1/n

limnP(1n1ni=1nXi1nn)=limnP(1ni=1nXi1)

=limnP(X¯n1)=limnFX¯n(1)=1

여기서 은 표본 평균 의 분포 함수이며 , LLN에 의해 ​​확률 (및 분포도)이 상수 수렴 되므로 마지막 평등입니다.FX¯n()X¯n1

따라서 결과가 상충됩니다. 어느 것이 맞습니까? 그리고 왜 다른 사람이 잘못 되었습니까?


1
@JuhoKokkala 물론입니다. 여기는 math.stackexchange.com/q/2830304/87400 OP의 실수를 무시하십시오.
Alecos Papadopoulos

2
문제는 LLN을 호출하는 두 번째 진술에 있다고 생각합니다.
Glen_b-복지국 Monica

3
나는 마지막 평등까지 당신을 따라 갔다. 우리가 기대할 수 있기 때문에, 분명히 잘못 P(X¯n1) 에 근접하는 1/2 대에 대한 n 과 동일하지 않아야 때문에 한계 1. 그것의 의도 정당성은 무엇입니까? 내가 아는 많은 법칙의 진술이 아닙니다.
whuber

1
@whuber 샘플 평균에 대한 모든 확률이 값 집중된다고 가정 합니다. 이것이 틀렸다면, 실수에 대한 답을 자세히 설명하는 것이 중요하다고 생각합니다. 이것이 바로이 질문의 목적입니다. 1
Alecos Papadopoulos 2016

2
Alecos, 내 우려는 마지막 단계가 잘못된 지 여부가 아닙니다 . 결국 질문에 관한 것이 아닌가? 나는 아직도 그 이유를 제시하는 당신에게서 아무것도 읽지 않았으며 그것이 무엇인지 추측하기조차 망설입니다. 귀하는 "LLN"을 언급하지만 귀하의 문제 해결은 귀하가 "LLN"을 이해한다고 주장하는 내용을 정확하게 설명하는 데 있다고 생각합니다.
whuber

답변:


15

이 오류는 여기에 다음과 같은 사실 가능성이 높습니다 : 유통의 융합은 암시 적으로 그 가정 로 수렴을 F ( X ) 에서 의 연속성의 점 F ( X ) . 한계 분포는 상수 랜덤 변수 이므로 x = 1 에서 점프 불연속성을 가지므로 CDF가 F ( x ) = 1로 수렴한다고 결론을 내릴 수 없습니다 . Fn(x)F(x) F(x)x=1F(x)=1


1
우리가 분산에서 수렴을 정의하는 방식 은 불연속 지점에서 수렴의 가능성을 배제 하지 않으며 단지 필요 하지는 않습니다 .
Alecos Papadopoulos 2018

1
유통의 융합이 필요하지 않습니다하지만 에 수렴 F ( 1 ) , 질문의 마지막 평등을 기반으로 무엇인가? Fn(1)F(1)
Juho Kokkala

1
@Juho 그것은 무엇을 기반으로하지 않습니다. 그것은 문제의 핵심입니다. 문제의 마지막 방정식을 만들 수있는 정리는 없습니다.
whuber

1
@AlecosPapadopoulos : 나는 그것이 가능성을 배제하지 않는다고 말한 적이 없다. 나는 분산의 수렴에서 당신에게 주어진 것 이상의 마지막 평등을 정당화해야한다고 암시 적으로 말하고 있습니다. 예를 들어 이 Bernoulli이면 사실입니다. Xn
Alex R.

11

IID 랜덤 변수 가진 E [ X I ] = VAR ( X I ) = 1 정의 Z의 NXiE[Xi]=var(Xi)=1 이제 CLT는 모든고정실수z에 대해limnFZn(z)=Φ(z-1)라고 말합니다. OP는 CLT를 적용하여LimnP(Zn1을 평가

Zn=1ni=1nXi,Yn=1ni=1nXi.
zlimnFZn(z)=Φ(z1)
limnP(Zn1n)=Φ(0)=12.

OP의 질문에 대한 다른 의견과 여러 의견이 지적했듯이 OP의 평가는 입니다. iid X i 가 확률이 1 인02 를 취하는 이산 랜덤 변수 인 특수한 경우를 고려하십시오.limnP(Yn1)Xi02 . 이제Σ N = 1 Xi가취할 수있는 모든에서도 정수 값[0,2N]등 때를N, 홀수 Σ N = 1 X값에 대해 수행 할 수없음그러므로 및YN=112i=1nXi[0,2n]ni=1nXin는 값1을취할 수 없습니다. 또한,Yn의 분포는약1이므로 대칭이므로 P(Yn1)=FYn(1)의값은1입니다.Yn=1ni=1nXi 1Yn1P(Yn1)=FYn(1)n이 홀수 때마다 2 따라서,시퀀스번호의 P(Y11),P(Y21),...,P(YN1),... 포함 된서브 시퀀스P(Y11),P가(Y31),,P(Y12n

P(Y11),P(Y21),,P(Yn1),
모든 항의 값이 1
P(Y11),P(Y31),,P(Y2k11),
. 한편,서브 시퀀스P(Y21),P(Y41),...,P(Y2K1),... 되는집광1. 따라서,limnP(Yn1)가 존재하지 않으며P의 수렴 주장(Yn1)12
P(Y21),P(Y41),,P(Y2k1),
1limnP(Yn1) 부터 1까지는 많은 의심으로보아야합니다.P(Yn1)

8

첫 번째 결과는 맞습니다. 다음과 같은 잘못된 설명에서 두 번째 부분에서 오류가 발생합니다.

limnFX¯n(1)=1.

이 진술은 허위입니다 (오른쪽은 1 이어야합니다) ) 그것은주장 된많은 수법칙을따르지 않습니다. 많은 수의 약한 법칙 (당신이 부르는)은 다음과 같이 말합니다 :12

limnP(|X¯n1|ε)=1for all ε>0.

ε>0|X¯n1|εX¯n1X¯n>1limnP(X¯n1)=1


1
(실제로 잘못된) 결과는 "확률의 수렴은 분포의 수렴을 의미한다"는 의미에서 비롯된다. 이 주장은 주장이 LLN에서 직접 온다는 것을 나타내지 않는다 .
Alecos Papadopoulos 2016

@AlecosPapadopoulos : 확률에 수렴 하지 분포의 융합을 의미한다. 다시, 분포의 수렴은 연속성의 지점에서만 요구된다. 그러나 아마도 수렴 확률이 포인트 의 분포 수렴을 의미하지는 않습니다 .
Alex R.

@AlexR. I am not sure where your objection lies. I believe this issue is covered in my own answer.
Alecos Papadopoulos

3

Convergence in probability implies convergence in distribution. But... what distribution? If the limiting distribution has a jump discontinuity then the limits become ambiguous (because multiple values are possible at the discontinuity).

where FX¯n() is the distribution function of the sample mean X¯n, which by the LLN converges in probability (and so also in distribution) to the constant 1,

This is not right, and it is also easy to show that it can not be right (different from the disagreement between CLT and LLN). The limiting distribution (which can be seen as the limit for a sequence of normal distributed variables) should be:

FX¯(x)={0for x<10.5for x=11for x>1

for this function you have that, for any ϵ>0 and every x, the difference |FX¯n(x)FX¯(x)|<ϵ for sufficiently large n. This would fail if FX¯(1)=1 instead of FX¯(1)=0.5


Limit of a normal distribution

It may be helpful to explicitly write out the sum used to invoke the law of large numbers.

X¯n=1ni=1nXiN(1,1n)

The limit n for X^n is actually equivalent to the Dirac Delta function when it is represented as the limit of the normal distribution with the variance going to zero.

Using that expression it is more easy to see what is going on under the hood, rather than using the ready-made laws of the CLT an LLN which obscure the reasoning behind the laws.


Convergence in probability

The law of large numbers gives you 'convergence in probability'

limnP(|X¯n1|>ϵ)=0

with ϵ>0

An equivalent statement could be made for the central limit theorem with limnP(|1n(Xi1)|>ϵn)=0

It is wrong to state that this implies

limnP(|X¯n1|>0)=0

It is less nice that this question is cross-posted so early (confusing, yet interesting to see the different discussions/approaches math vs stats, so not that too bad). The answer by Michael Hardy on the math stackexchange deals with it very effectively in terms of the strong law of large numbers (the same principle as the accepted answer from drhab in the cross posted question and Dilip here). We are almost sure that a sequence X¯1,X¯2,X¯3,...X¯n converges to 1, but this does not mean that limnP(X¯n=1) will be equal to 1 (or it may not even exist as Dilip shows). The dice example in the comments by Tomasz shows this very nicely from a different angle (instead of the limit not existing, the limit goes to zero). The mean of a sequence of dice rolls will converge to the mean of the dice but the probability to be equal to this goes to zero.


Heaviside step function and Dirac delta function

The CDF of X¯n is the following:

FX¯n(x)=12(1+erfx12/n)

with, if you like, limnFX¯n(1)=0.5 (related to the Heaviside step function, the integral of the Dirac delta function when viewed as the limit of normal distribution).


I believe that this view intuitively resolves your question regarding 'show that it is wrong' or at least it shows that the question about understanding the cause of this disagreement of CLT and LLN is equivalent to the question of understanding the integral of the Dirac delta function or a sequence of normal distributions with variance decreasing to zero.


2
Your limiting distribution is in fact not a distribution at all. A CDF must be right continuous, whereas it clearly is not at x=1/2.
Alex R.

The right continuity seems to be necessary such that for every a we have limnFX(a+1n)=FX(a) as the events Xa+1n are nested we should have
limnFX(a+1n)=limnP(Xa+1n)=P(limnXa+1n)=P(Xa)=FX(a)
but is this true for our case and where is the catch? Is this right continuity necessary based on probability axioms or is it just a convention such that the CDF works for most common cases?
Sextus Empiricus

@Martin Weterings: This is precisely where it comes from. Any valid measure P must satisfy these monotonicity results. They are a consequence of the boundedness of P along with countable additivity. More generally, a function F(x) is a CDF (i.e. corresponds to some distribution P via F(b)F(a)=P(a<Xb) iff F is right-continuous, along with being monotonic, and having left limit 0, right limit 1.
Alex R.

2

I believe it should be clear by now that "the CLT approach" gives the right answer.

Let's pinpoint exactly where the "LLN approach" goes wrong.

Starting with the finite statements, it is clear then that we can equivalently either subtract n from both sides, or multliply both sides by 1/n. We get

P(1ni=1nXin)=P(1ni=1n(Xi1)0)=P(1ni=1nXi1)

So if the limit exists, it will be identical. Setting Zn=1ni=1n(Xi1), we have, using distribution functions

P(1ni=1nXin)=FZn(0)=FX¯n(1)

...and it is true that limnFZn(0)=Φ(0)=1/2.

The thinking in the "LLN approach" goes as follows: "We know from the LLN that X¯n converges in probabililty to a constant. And we also know that "convergence in probability implies convergence in distribution". So, X¯n converges in distribution to a constant". Up to here we are correct.
Then we state: "therefore, limiting probabilities for X¯n are given by the distribution function of the constant at 1 random variable",

F1(x)={1x10x<1F1(1)=1

... so limnFX¯n(1)=F1(1)=1...

...and we just made our mistake. Why? Because, as @AlexR. answer noted, "convergence in distribution" covers only the points of continuity of the limiting distribution function. And 1 is a point of discontinuity for F1. This means that limnFX¯n(1) may be equal to F1(1) but it may be not, without negating the "convergence in distribution to a constant" implication of the LLN.

And since from the CLT approach we know what the value of the limit must be (1/2). I do not know of a way to prove directly that limnFX¯n(1)=1/2.

Did we learn anything new?

I did. The LLN asserts that

limnP(|X¯n1|ε)=1for all ε>0

limn[P(1ε<X¯n1)+P(1<X¯n1+ε)]=1

limn[P(X¯n1)+P(1<X¯n1+ε)]=1

The LLN does not say how is the probability allocated in the (1ε,1+ε) interval. What I learned is that, in this class of convergence results, the probability is at the limit allocated equally on the two sides of the centerpoint of the collapsing interval.

The general statement here is, assume

Xnpθ,h(n)(Xnθ)dD(0,V)

where D is some rv with distribution function FD. Then

limnP[Xnθ]=limnP[h(n)(Xnθ)0]=FD(0)

...which may not be equal to Fθ(0) (the distribution function of the constant rv).

Also, this is a strong example that, when the distribution function of the limiting random variable has discontinuities, then "convergence in distribution to a random variable" may describe a situation where "the limiting distribution" may disagree with the "distribution of the limiting random variable" at the discontinuity points. Strictly speaking, the limiting distribution for the continuity points is that of the constant random variable. For the discontinuity points we may be able to calculate the limiting probability, as "separate" entities.


The 'lesson learned' perspective is interesting, and this is a good, not too difficult, example for didactic application. Although I wonder what kind of (direct) practical application this thinking about the infinite has, because eventually in practice n
Sextus Empiricus

@MartijnWeterings Martijn, the motivation here was certainly educational, a) as an alert to discontinuities even in such a "flat" situation as the convergence to a constant, and so also in general (they destroy uniform convergence for example), and b) a result on how the probability mass is allocated becomes interesting when the sequence that converges in probabilty to a constant, still has a non-zero variance.
Alecos Papadopoulos

We could say that CLT let's as say something about convergence to a limiting normal distributed variable (thus being able to express such things as F(x)), but LLN only allows us to say that, by increasing the sample size, we get closer to the true mean, but this does not say that we get, with higher probability, 'exactly equal to the sample mean'. LLN means that the sample mean gets closer and closer to a limiting value but not (with higher probability) equal to it. LLN says nothing about F(x)
Sextus Empiricus

The original thoughts around the LLN where actually opposite (see the reasoning of Arbuthnot stats.stackexchange.com/questions/343268). "It is visible from what has been said, that with a very great Number of Dice, A’s Lot would become very small... there would be but a small part of all the possible Chances, for its happening at any assignable time, that an equal Number of Males and Females should be born."
Sextus Empiricus
당사 사이트를 사용함과 동시에 당사의 쿠키 정책개인정보 보호정책을 읽고 이해하였음을 인정하는 것으로 간주합니다.
Licensed under cc by-sa 3.0 with attribution required.